Last visit was: 24 Apr 2024, 20:42 It is currently 24 Apr 2024, 20:42

Close
GMAT Club Daily Prep
Thank you for using the timer - this advanced tool can estimate your performance and suggest more practice questions. We have subscribed you to Daily Prep Questions via email.

Customized
for You

we will pick new questions that match your level based on your Timer History

Track
Your Progress

every week, we’ll send you an estimated GMAT score based on your performance

Practice
Pays

we will pick new questions that match your level based on your Timer History
Not interested in getting valuable practice questions and articles delivered to your email? No problem, unsubscribe here.
Close
Request Expert Reply
Confirm Cancel
SORT BY:
Date
Tags:
Show Tags
Hide Tags
Current Student
Joined: 04 Feb 2014
Posts: 186
Own Kudos [?]: 568 [39]
Given Kudos: 164
Location: India
Concentration: General Management, Entrepreneurship
GPA: 3
WE:Project Management (Manufacturing)
Send PM
Most Helpful Reply
GMAT Club Legend
GMAT Club Legend
Joined: 12 Sep 2015
Posts: 6821
Own Kudos [?]: 29916 [9]
Given Kudos: 799
Location: Canada
Send PM
General Discussion
Retired Moderator
Joined: 26 Nov 2012
Posts: 473
Own Kudos [?]: 493 [4]
Given Kudos: 46
Send PM
Intern
Intern
Joined: 12 Jun 2016
Posts: 22
Own Kudos [?]: 17 [0]
Given Kudos: 18
Location: India
Concentration: Entrepreneurship, Finance
GPA: 3.76
WE:Information Technology (Computer Software)
Send PM
Re: Given that x 5, is x > 1/(x - 5)^2 ? (1) x > 0 (2) x > 10 [#permalink]
Given Eq: x > 1/(x-5)^2

-> Multiply both sides by (x-5)^2 as it is always positive.
-> Equation becomes : x(x-5)^2 > 1

Statement 1: x>0
x can be any positive number. Say 5.05, this will make x(x-5)^2 less than 1.
But, putting x = 2,3,7..., this will make x(x-5)^2 greater than 1.
So Insufficient.
Statement 2: x>10
Putting x > 10 will always give x(x-5)^2 > 1. Hence, sufficient.

Ans: B
VP
VP
Joined: 10 Jul 2019
Posts: 1392
Own Kudos [?]: 542 [0]
Given Kudos: 1656
Send PM
Re: Given that x 5, is x > 1/(x - 5)^2 ? (1) x > 0 (2) x > 10 [#permalink]
Any other strategy to answer the question aside from number plugging?

Or is this question a prime candidate for number plugging

Posted from my mobile device
Intern
Intern
Joined: 07 Jul 2019
Posts: 22
Own Kudos [?]: 3 [0]
Given Kudos: 19
Location: India
Schools: Ivey '24 (M$)
GMAT 1: 700 Q50 V34
Send PM
Re: Given that x 5, is x > 1/(x - 5)^2 ? (1) x > 0 (2) x > 10 [#permalink]
@e-gmat Can you provide a solution to this question in the wav@y line method approach?
Manager
Manager
Joined: 10 Jul 2021
Posts: 233
Own Kudos [?]: 48 [1]
Given Kudos: 29
Send PM
Re: Given that x 5, is x > 1/(x - 5)^2 ? (1) x > 0 (2) x > 10 [#permalink]
1
Kudos
Fdambro294 wrote:
Any other strategy to answer the question aside from number plugging?

Or is this question a prime candidate for number plugging

Posted from my mobile device


1) x>0
x-5>-5
(x-5)^2>0 (please check why)
x>0
=>
x(x-5)^2>0
now put x=1/100 then it is <1 and when x=25 it is >1.

2) x>10
x-5>5
(x-5)^2>25
x>10
x*(x-5)^2>250>1
hence b is good.
Manager
Manager
Joined: 01 Jan 2014
Posts: 204
Own Kudos [?]: 69 [1]
Given Kudos: 420
Location: United States (MI)
Send PM
Re: Given that x 5, is x > 1/(x - 5)^2 ? (1) x > 0 (2) x > 10 [#permalink]
1
Kudos
BrentGMATPrepNow wrote:
anurag16 wrote:
Given that x ≠ 5, is x > 1/(x-5)²

Statement #1: x > 0

Statement #2: x > 10


Target question: Is x > 1/(x-5)² ?
This is a great candidate for rephrasing the target question.
Since (x-5)² is guaranteed to be POSITIVE, we can take the inequality x > 1/(x-5)² and multiply both sides by (x-5)²
When we do this, we get: (x)(x-5)² > 1

So, we can REPHRASE the target question as....
REPHRASED target question: Is (x)(x-5)² > 1?

Statement 1: x > 0
This statement doesn't FEEL sufficient, so I'll TEST some values.
There are several values of x that satisfy statement 1. Here are two:
Case a: x = 10, in which case (x)(x-5)² = (10)(10-5)² = 125. In this case, (x)(x-5)² > 1
Case b: x = 0.01, in which case (x)(x-5)² = (0.01)(0.01-5)² ≈ (0.01)(25) ≈ 0.25 In this case, (x)(x-5)² < 1
Since we cannot answer the REPHRASED target question with certainty, statement 1 is NOT SUFFICIENT

Aside: For more on this idea of plugging in values when a statement doesn't feel sufficient, you can read my article: https://www.gmatprepnow.com/articles/da ... lug-values

Statement 2: x > 10
If x is greater than 10, it is clear that (x)(x-5)² MUST be greater than 1
Since we can answer the REPHRASED target question with certainty, statement 2 is SUFFICIENT

Answer =

Related Videos






BrentGMATPrepNow, Bunuel - How would I know I have missed a data point and how to select data points to arrive at the correct answer? I never thought of, for instance, plugging in 0.00001. I simply checked for integer and a fraction value (0.5 in my case) and obviously got the wrong answer. Thanks for your response.
CR Forum Moderator
Joined: 25 Jan 2022
Posts: 832
Own Kudos [?]: 643 [0]
Given Kudos: 558
Location: Italy
GPA: 3.8
Send PM
Re: Given that x 5, is x > 1/(x - 5)^2 ? (1) x > 0 (2) x > 10 [#permalink]
never mind made some mistakes =)
GMAT Club Bot
Re: Given that x 5, is x > 1/(x - 5)^2 ? (1) x > 0 (2) x > 10 [#permalink]
Moderator:
Math Expert
92900 posts

Powered by phpBB © phpBB Group | Emoji artwork provided by EmojiOne